« first day (13 days earlier)      last day (561 days later) » 
00:00 - 16:0016:00 - 00:00

12:04 AM
0
Q: Find the maximum area of a right triangle with a constant perimeter P.

chuckster20I have been learning calculus from a tutor and I have been trying to solve a problem that he gave me. The problem is to find the maximum area of a right triangle with a constant perimeter [;P;]. To start solving this problem I wrote down the different equations for the area and perimeter of a rig...

Welcome to Math.SE, chuckster20. This site uses MathJax formatting of formulas. More tips at: How to ask a good question? (autocomment)Normal Human 21 secs ago
 
12:21 AM
0
Q: Is the set closed?

Jennie Durham Suppose that $U$ is closed. and $V=\Bbb R^n \setminus U$. Now, is V closed or neither open nor closed? Thanks in advance!

Short question.
0
Q: Proof involving an $\iff$ statement related to eigenvalues and eigenvectors

nbroI have the following theoreom: Let us consider the matrix $A\in\mathbb{R}^{n,n}$. $\lambda$ is an eigenvalue of $A$ iff $(\lambda I - A)$ is not invertible, or, equivalently, $\det (\lambda I - A) = 0$. I managed to prove the part $\Rightarrow$, but not the other way around, i.e. given that...

Tag (proof-writing) should not be the only tag a question has. Please add a tag for a subject area to which the question belongs. (autocomment)Normal Human 21 secs ago
 
12:49 AM
0
Q: sum over a function at random times = integral?

korziI got a markov kernel $\sigma_x(A)$ and arrival times of a Poisson process $\Gamma_i$. I need to calculate the sum \begin{align} \sum_{i=1}^\infty \sigma_{\Gamma_i} (A). \end{align} I need to get \begin{align} \int_0^\infty\sigma_r(A)dr, \end{align} but i have no idea why both of them should b...

Body punctuation.
0
Q: how can I show that a line in an edge of a polytope is a boundary point of the polytope (

najiShow that line in an edge of a polytope is boundary point of the polytope

Short question.
 
-1
Q: AJAX Notification Alets

MortuusStack overflow would be better if notifications of activity were displayed to the user asynchronously. All you would need to do is alter your template. At the top, the Recent Inbox Messages button. Funny, as I was typing this, the requested feature seems to have happened. Still, I often click bac...

 
1:15 AM
0
Q: Proof involving neighborhood of a limit point

Ryan T. DonnellyLet $f:D \to R$ with $x_0$ as an accumulation point of $D$. Prove that $f$ has a limit at $x_0$ if for each $\epsilon >0$ there is a neighborhood $Q$ of $x_0$ such that, for an $x,y \in Q \cap D$, $x \ne x_0$ and $y \ne x_0$, we have $\lvert f(x)-f(y) \rvert < \epsilon$. Pf:Let $x_0$ be an accu...

Tagged proof-verification.
0
Q: determinant of skew-symmetric matrix

JonnyHelp me, please, to solve this task: Prove that the determinant of skew-symmetric matrix of even order have a full square as a polynomial in matrix elements

Short question.
0
Q: Sequneces space

Sanwar$l_\infty$ is the space of bounded sequences and $c_0$ is the space of sequences converge to 0, is a closed subspace of $l_infty$. I am trying to prove that for any $x \in l_\infty$, $d(x,c_0) = \limsup_{n\to \infty} |x_n|$.

Welcome to Math.SE, Sanwar. Consider replacing (analysis) with a more specific tag for the relevant branch of analysis. (autocomment)Normal Human 21 secs ago
0
Q: Convergence/Divergence of Series for e^-1

GaramMasalaHow do I show that the series $\sum \frac{n^n}{n!} x^n$ diverges at $x =e^{-1}$? I understand that $a(n) = (\frac{n}{n+1})^n$ is a strictly decreasing function and therefore $(\frac{n}{n+1})^n > \frac{1}{e}$. Hope it helps.

Title contains 1.
 
1:34 AM
0
Q: Need help understanding proof (Munkres)

AvatrinThere's a step in a proof in Munkres' Topology that doesn't make sense to me. Theorem: Let A be a connected subspace in X. If $A \subset B \subset \bar{A}$, then B is also connected. Proof: Let A be connected, and $A \subset B \subset \bar{A}$. Suppose that $B = C \cup D $ is a separation of B....

Words such as help do not add information to titles. Please edit the title so that it better describes the specifics of your question. Do not hesitate to make it longer or include a formula if needed. More tips at: How to ask a good question? (autocomment)Normal Human 21 secs ago
 
1:54 AM
0
Q: Show that there exist Borel sets $B_n$ with some properties

timon92Let $X$ be a Polish space. Let $B$ be a Borel subset of $X \times X$ with the following property: $$\forall x \in X \ \left|\left\{y : (x,y) \in B\right\}\right| \le\aleph_0.$$ Show that there exists a sequence of Borel sets $B_0, B_1, \dots, B_n, \ldots \subset X \times X$ such that $B = \bigcu...

Consider adding a tag for a broader subject area to which the question belongs. Some of these tags might fit. (autocomment)Normal Human 21 secs ago
0
Q: Why not, first order logic to DNF conversion?

DanielThere seems to be huge amount of discussion about converting "first order logic to CNF". But don't see much about "first order logic to DNF" conversion. What is the reason?

Short question.
0
Q: Finding probability distribution of X

Andrew NunnoThe joint probability density function of X and Y is given by f(x,y)=(15/25000)(xy+y^2) if y^2−25≤x≤−y^2+25 and f(x,y)=0 otherwise. Whenever I integrate for Y disregarding x I don't get a probability distribution of X that then integrates to 1. I believe that x has to be between -25 and 25 with...

0
Q: How to solve this integral using substitution?

Leon MaHere is the integral question I know it might not be so hard, but I just cannot think of a way to solve it.

Short question.
0
Q: Geometric Series - absolutely convergent, conditionally convergent or divergent?

PierreHow would I determine if the following series is absolutely convergent, conditionally convergent or divergent? $\sum\limits_{i=1}^n$$\sqrt[n]{2}+1$

Short question.
0
Q: How should I approach the problem where I need to prove $\lim _{ x\rightarrow \infty }{ \sqrt { f(x) } =0 } $?

nikolitaLet $f$ be a positive function defined on an interval $[a, \infty)$, such that $\lim _{ x\rightarrow \infty }{ { f(x) } =0 } $ ?. Prove that $\lim _{ x\rightarrow \infty }{ \sqrt { f(x) } =0 } $ ? I can't seem to be able to come up with any way to approach this proof. A hint to put me in the r...

Title contains problem.
0
Q: For every two real numbers x and y, there is some real number z such that xz=y

Ploni AlmoniWas just looking at this question, just would like to make sure the proof is sound. is the following statement true? $∀x,y ∈ R:∃z ∈ R$ such that $xz=y$ so immediately I saw because $x$ ,$y$ ,and $z$ are real numbers I knew that the statement was true. I took it further and demonstrated a proof...

Tagged proof-verification.
0
Q: shoting with prabability game theory

User00012 people standing in front of each other in a rail with distance of 2 meters in which player A stand on point -1 and player B stand on point 1. They have only one gun with one bullet. Player A can fire on points -1 or -0.5 or 0 (it is intuitive that he could do it if be alive in points -0.5, 0)...

Welcome to Math.SE, User0001. This site uses MathJax formatting of formulas. More tips at: How to ask a good question? (autocomment)Normal Human 21 secs ago
 
2:23 AM
0
Q: How to prove limit the truncated random variables

ScottLet $\alpha>0$ and define the truncated random variables $X_n^{a}=X_n$ if $X_n<a$, and $X_n^{a}=a$ if $X_n\geq a$.How to prove $E(X_n^{a})\rightarrow E(X_n)$ as $a\rightarrow\infty$ ?

Short question.
 
2:41 AM
0
Q: Find the set of real numbers $u$ such that $u/(u−1) ≤ 2$.

Ploni AlmoniFind the set of real numbers $u$ such that $u/(u−1) ≤ 2$. I don't really know how to get started with this. Thanks

Short question.
0
Q: Hi, this is a chess question concerning how to tell who is the stronger player. It has application in fine tunning chess engines.

David Eatherthis is a chess like question concerning how many games are required to tell who is the stronger player. The application is in fine tuning chess engines. It is typical that during this process a chess engine will play some games with a modified version of itself. After the tests the most succes...

Welcome to Math.SE, David Eather. This site uses MathJax formatting of formulas. More tips at: How to ask a good question? (autocomment)Normal Human 21 secs ago
 
3:12 AM
0
Q: How can I prove the following question.

R.Rojasenter image description here HELP

Welcome to Math.SE, R.Rojas. Words such as question do not add information to titles. Please edit the title so that it better describes the specifics of your question. Do not hesitate to make it longer or include a formula if needed. More tips at: How to ask a good question? (autocomment)Normal Human 21 secs ago
0
Q: Question about a Topic in Statistics/ Optimization

GidyHi guys I was reading about this topic Maximum Likelihood degree. I am not very familiar with statistics and the maximum likelihood estimate, but I want to learn what is the Maximum Likelihood degree and I have read some literature http://arxiv.org/pdf/math/0406533.pdf I think I can even follow...

Words such as question do not add information to titles. Please edit the title so that it better describes the specifics of your question. Do not hesitate to make it longer or include a formula if needed. More tips at: How to ask a good question? (autocomment)Normal Human 22 secs ago
0
Q: Primitive cohomology, example request

Feng Hao$X$ is a compact K$\ddot{a}$hler manifold or smooth projective variety. is there an example that a primitive class of $H^*(X, \mathbb{C})$ is wedge product of other two primitive classes?

Short question.
0
Q: question about extreme value distribution

Chen xiaojiaI am studying probability and finding hard to understand the following equation. $$\pi_i=Pr[P_i(j) \leq \min\{P_s(j);s \neq i\}]=\int_0^\infty \underset{s\neq i}{\prod}[1-G_s(p)]dG_i(p)=\frac{T_i(c_id_{i})^{-\theta}}{\sum T_i(c_id_{i})^{-\theta}},$$ where $G_i(p)=Pr[P_i \leq p]$ and $G_s=1-e^{-T...

Words such as question do not add information to titles. Please edit the title so that it better describes the specifics of your question. Do not hesitate to make it longer or include a formula if needed. More tips at: How to ask a good question? (autocomment)Normal Human 21 secs ago
0
Q: What is the best approximate of points on a sphere?

remoI have a unit raduis sphere with $n$ point on it. What is the best representation of these points in a 4 dimensional space for preserving the original distances? Your help is appreciated.

Short question.
 
3:29 AM
0
Q: Why are some questions not getting so many views?

SweeperI often have problems programming and I always post questions. I see some of my questions get a lot of views while the others did not. I know that more people will look at questions with popular tags such as java, c# and android. But my popular questions have roughly the same tags as the ones th...

 
0
Q: Help detecting an error

user12344321I apologize if the error is easily spotted. But, here it is: If we are computing $\int^1_0{\frac{x^2-y^2}{(x^2+y^2)^2}}dy$ and use the substitution $y\rightarrow x \cdot tan(\theta)$ to get $$\int_0^{\pi/4}\frac{x^2-x^2tan^2(\theta)}{(x^2+x^2tan^2(\theta))^2}sec^2(\theta)d\theta=\frac{1}{x^2}\i...

Words such as help do not add information to titles. Please edit the title so that it better describes the specifics of your question. Do not hesitate to make it longer or include a formula if needed. More tips at: How to ask a good question? (autocomment)Normal Human 21 secs ago
 
3:40 AM
0
Q: has anyone used 'the way of analysis' by strichartz?

Mathchothe reviews of the book on the amazon say this one is detailed and good. i once took analysis course and want to study materials again. so would this book be good for my purpose to review analysis on my own? or just baby rudin suffices? any advice will be appreciated.

Tag (soft-question) should not be the only tag a question has. Please add a tag for a subject area to which the question belongs. (autocomment)Normal Human 21 secs ago
0
Q: Evaluating a regular expression to a minimal expression

darkserithEvaluate to a minimal expression: b* a* ∩ a* b* To me, the only elements to both sets are the empty string, strings containing only a, and strings containing only b, so isn't the answer just a* ∪ b* ? This was a midterm problem and the prof marked the answer incorrect. Please help. Thanks.

 
0
Q: two similar questions about mentioning GRE score in CV for grad applications

aparente001Can it hurt to put my GRE score on my resume? Put good GRE scores on resume submitted with grad app? I'm not sure what to suggest.

 
4:05 AM
0
Q: Rotation and angular speed problem

fleech I am having problems with a). I figured out that the angle is 87.48 degrees. And the length of an arc C (the displacement) was found to be 3.35899 rad

Title contains problem. Short question.
 
4:21 AM
0
Q: How do conjugation rules apply for vectors in complex spaces? So I have (1,0,0),(1,2,0),(1,2,3)

user289419Inner product is defined to be x1*conjugate of x2 + y1*conjugate of y2 +z1*conjugate of z2 Is the inner product of (1,0,0)(1,2,0), 1*1 or 1*-1? Also I have the complex vector space with span ((1,1,0,0),(1,1,1+i,1+i),(1 + i,1+i,i,i),(1,1,1,1)). I know that they are LI since the only combination ...

 
4:33 AM
0
Q: Data Management Frequency Interval

user164403I am given the results of 30 students for a Data Management test: $78, 81, 55, 60, 65, 86, 44, 90, 59, 81, 77, 71, 65, 39, 80, 72, 70, 64, 68, 78, 88, 73, 61, 70, 75, 96, 51, 73, 65, 67.$ The question I'm confused with goes as following: a) Determine the range for this data. That's easy - I su...

Questions tend to get more attention when they have a tag for a broad area of mathematics relevant to the question. Some of these tags might fit. (autocomment)Normal Human 20 secs ago
0
Q: Meaning of $\int y dx +x dy$

thats_how_i_feelWhat does it mean to take an integral: $\int y dx + xdy$ I would guess that this means that this integral over a region multiplies the infinitely small increments of $x$ ($\Delta x$) by $y$ and the infinitely small increments of $y$ ($\Delta y$) by x, but I still can't really picture what this i...

Short title.
0
Q: Understanding this ring and isomorphisms

QualityI am trying to make sense of what the following ring really is. I have that $$\mathbb{Q}(\epsilon)=\mathbb{Q}[x]/x^2$$ where $\epsilon$ denotes the coset of $x$ in the quotient ring. So I know elements of $\mathbb{Q}(\epsilon)$ can be written as $a+b\epsilon$ where $a$ and $b$ $\in \mathbb{Q}$ ...

Question contains please, please. Tagged proof-writing.
0
Q: An application of the Inverse function theorem

BLAZEI have recently come across two formula's that I am unfamiliar with and would like to know if they are both aspects of the same thing: $$\color{purple}{\cfrac{1}{f^{\prime}(a)}=f(a)(f^{-1})^{\prime}}\tag{1}$$ $$\rho_x (x)=\rho_\alpha(\alpha)\left|\frac{\mathrm{d}x}{\mathrm{d}\alpha}\right|^{-1}...

Consider adding a tag for a broader subject area to which the question belongs. Some of these tags might fit. (autocomment)Normal Human 21 secs ago
0
Q: Find solutions for x and y using Eulers formula

J.Doee^xi=x/(1-i)+y/(2+3i) Find all solutions for x and y `

Short question.
0
Q: reciprocal-of-a-reciprocal proof

sam.bishop(My question is similar to this one at a high level, but I am looking for something more rigorous.) I have started into Michael Spivak's "Calculus" textbook. Problem 3 (v) on page 14 asks for a proof that "$\frac{a}{b} \big/\frac{c}{d} = \frac{ad}{bc}$, if b, c, d $\neq 0$". The only proof tha...

Tag (proof-explanation) should not be the only tag a question has. Please add a tag for a subject area to which the question belongs. (autocomment)Normal Human 21 secs ago
0
Q: Proving the limit of a radical using epsilon delta

Ryan T. DonnellyDefine $f:(0,1)->R$ by $f(x)=\frac{\sqrt{9-x}-3}{x} $ I know that $f(x)=\frac{\sqrt{9-x}-3}{x}*\frac{\sqrt{9-x}+3}{\sqrt{9-x}+3}=\frac{9-x-9}{\sqrt{9-x}+3}=-\frac{1}{\sqrt{9-x}+3} $ Since $f$ is only defined in $(0,1)$, $-\frac{1}{5}<-\frac{1}{\sqrt{9-x}+3}<-\frac{1}{6} $ So can I just pick so...

Tagged proof-verification.
0
Q: Is it enough to show a sequence converge to infinity if the limit of this sequence is greater or equal infinity?

user81235Suppose I derive $\lim_{n\rightarrow\infty}{a_n}\geq\lim_{n\rightarrow\infty}b_{n}=\infty$. Can I conclude $\lim_{n\rightarrow\infty}a_n=\infty$?

Short question.
0
Q: Differentiation of $(x)(x+y)$

user963241I checked the differentiation of $(x)(x+y)$ using an online derivative tool which gives the result: $\frac{d}{dx}\left(\left(x\right)\left(x+y\right)\right) = x+y+x\left(\frac{d}{dx}\left(y\right)+1\right)$ But using a different tool I found that derivate is: $\frac{d}{dx}\left(\left(x\right)\...

Short title.
0
Q: How to find the percent variation in Y is explained by X?

FrankI know that the r^2 value for the data is 0.9832. Is there a way to use that value to find the percent variation in Y is explained by X? Or do I need to use the data given to me?

Short question.
0
Q: Where am I going wrong in my gram schmit steps? Can't seem to get the right answer

user289419So I have the vectors (1,0,0),(1,2,0),(1,2,3). $v_1$=(1,0,0) which also has norm 1 $v_2$=(1,2,0)-$\frac{<(1,2,0)(1,0,0)>}{1}$*(1,0,0) = (0,2,0) $v_3$=(1,2,3)-$\frac{<(1,2,3)(1,0,0)>}{1}$*(1,0,0)- $\frac{<1,2,3)(0,2,0)>}{2}$*(0,2,0) =(0,-2,3) Now the answer is (1,0,0),(0,1,0),(0,0,1), I hav...

Title contains wrong.
0
Q: How to use the Set Builder Notation to express the composite relation R^n?

Jasmine LatendresseThis is what I came up with R^n={(x,z) ∈ R^n |∃y,(x,y) ∈ R^n ∧ (y,z) ∈ R^n} It's just that the R^n bugs me a lot...Can someome explain? I have read the book and look through the web but I don't really get it.

0
Q: Is it possible to upgrade the statement that an unramified finite map is a "cover" to the statement that a ??? map is a fiber bundle?

AreaManI am thinking of the following theorem: All varieties are projective over algebraically closed fields. Let $f: X \to Y$ be an unramified finite map to a nonsingular variety $Y$. Then $f$ is locally described as the projection to $Y$ of a subvariety $X \subset Y \times A^1$, where $X$ is defined...

Title contains ??.
0
Q: Open and Closed Set intervals

R. MonksWhat is an example of an open set $A_1$ in the reals which contains the interval (1,2) but so that $A_1$ is not itself an interval. Find another $A_1$ except this time $A_1$ is a closed set containing (1,2) but is not itself an interval. I think that the open set $A_1$ would be something like...

Short title.
0
Q: Show that a surface is minimal - scherk

Ana HiaHave to show that first scherk surface is minimal : We have f(x,y)=(x,y,ln(cosx/cosy)) I am trying to show that using H=1/2(k+k')=0 (this is the definition given in the course) k and k' the principal curvatures but I don't get 0. Calculating the second fundamental form : fx=(1,0,-tanx) fxx=(...

Welcome to Math.SE, Ana Hia. This site uses MathJax formatting of formulas. More tips here. (autocomment)Normal Human 21 secs ago
0
Q: Find all the triples (a,b,c)

Sarnavo SarkarFind all the triples (a,b,c) of positive integer such that $(1+1/a)(1+1/b)(1+1/b)$ Where a,b,c can be distinct

Short title. Short question.
Short question. Tagged proof-writing.
 
5:40 AM
0
Q: How to generate random noise

Harutaka KawamuraI'm studying optical fibers and trying to analyze how the core radius fluctuations along the fiber length affect the performance of optical fibers. How can I generate random noise dr which satisfies the following equations? enter image description here

Welcome to Math.SE, Harutaka Kawamura. Questions tend to get more attention when they have a tag for a broad area of mathematics relevant to the question. Some of these tags might fit. (autocomment)Normal Human 20 secs ago
0
Q: Manipulating Compound Inequality

Ayush BhandariIf 1>1-ab>0, which of the following must be true? (1) a/b >0 (2) a/b <1 (3) ab <1 Option 1 and 3 are correct, but is option 2 necessarily correct?

Short title. Short question.
 
0
Q: Error message when leaving role blank is quite difficult to read

ZirakAs always, a beautiful picture is worth more than a thousand words, especially those which can not br read. .message.message-error has a color: white, which may be fitting for other places, but not quite here.

 
0
Q: Is there any continuous surjective map from $[0,1) $ to $\mathbb R$?

learnmoreIs there any continuous surjective map from $[0,1) $ to $\mathbb R$? If it exits then what is the way to construct such a map?

Short question.
0
Q: Convert 3^n to some form of 2^n

Shahbaz Ahmad SahiI am not from Maths field, but I need your help to convert the 3^n form to 2^n. I need to change the base from 3 to 2. The resultant expression can be of any form.

Short question.
 
0
Q: Can Stack Overflow and Meta's logos be changed temporarily to a Support For Paris logo?

Tom DworzanskiOn June 26, 2015, a StackOverflow user requested the logo be changed temporarily to show support for a then-happening situation. The request was granted. The logo was altered on many websites. A similar show of support is now happening for the victims of terrorism in France. Many other websites ...

 
6:02 AM
0
Q: Determine the minimal polynomial of $T$ with respect to $\begin{pmatrix} 0 \\ 0 \\ 1 \end{pmatrix}.$

George DillaLet $T: \mathbb{R}^3 \to \mathbb{R}^3$ be defined by $T(v)=\begin{pmatrix} 2 & -1 & 1 \\ -3 & 4 & -5 \\ -3 & 3 & -4 \end{pmatrix}v.$ Determine the minimal polynomial of $T$ with respect to $\begin{pmatrix} 0 \\ 0 \\ 1 \end{pmatrix}.$ $T(v)= \begin{pmatrix} 2 & -1 & 1 \\ -3 & 4 & -5 \\ -3 & 3 & -...

Title contains \begin, pmatrix, pmatrix.
0
Q: To confirm the Novikov's condition

sharpeI have a question about Novikov's condition. Let $L$ be a local martingale such that either $\exp \left(\frac{1}{2}L \right)$ is a submartingale or $E[\exp\left(\frac{1}{2} \langle L,L \rangle_{t} \right)]<\infty$ for every $t>0$. Then $\mathcal{E}(L)_{t}:=\exp \left\{L_{t}-\frac{1}{2} \langle ...

Question contains please, thank you.
Short question.
0
Q: Remainders & the GRE: Finding a remainder based on divisors.

Broguen WhetstoneI'm currently studying for the GRE and a specific type of question has me stumped. I have a two part question, but first here is one problem and my work: "If x is the remainder when a multiple of 4 is divided by 6, and y is the remainder when a multiple of 2 is divided by 3, what is the greatest...

Title contains gre.
0
Q: Factorizing Gaussian Integers

Jon Mark PerryIt is wellknown that $n=ab\in\mathbb{Z}, a,b\in\mathbb{Z}, a,b\gt1 \implies a\le \sqrt n \lor b\le\sqrt n$. Let $z=a+ib$ be a nonprime Gaussian integer, such that $z=(c+id)(e+if)$. Do we have that either $|c+id|\le\sqrt{|a+ib|}$ or $|e+if|\le\sqrt{|a+ib|}$? I am confused here because Gaussian ...

Short title.
0
Q: is minimum of two metrics is again a metric??

Naveen malagiLet $d1$ and $d2$ be two metrics on non empty set $X$ is $d$ = $/min{d1, d2}/$ is agai metric on $X$. M looking for a counter example for minimum of two metrics is not a metric ...help me out!!

Title contains ??.
0
Q: Can you get 21 with 9 and 10?

ShahsOkay, so everyone knows the joke that has been going on. "What's 9 + 10? 21". Yeah, so I was wonderning an you get 21 with 9 and 10 at all, this means any mathematical aspect can be used.

Short title. Short question.
 
6:30 AM
0
Q: What to do when someone deliberately down-votes many of the questions which I have asked?

Shirgill Farhan AnsariSomeone deliberately downvoted the questions which I have asked in the nearby past. The reason is that I downvoted him on one of my thread in biology.stackexchange.com. I came to know because I decently left a comment describing what was wrong with his answer. In fact his/her answer was remov...

 
6:44 AM
0
Q: Is $\mathbb Z_5[i]$ infinite?

operLet $\mathbb R=\mathbb Z_5[i]$ denote all polynomials $a_0+a_1i+a_2i^2+...+a_ki^k$ for any nonnegative k. What I got so far is that the polynomial is actually $a_0+a_1i-a_2-a_3i+a_4+a_5i+...$ which is $a_{4k}-a_{4k+2}+(a_{4k+1}-a_{4k+3})i$, which is in the form of $a+bi$ where $a,b\in \mathbb Z_...

Short title.
 
7:00 AM
0
Q: Finding the marginal distributions of a binosmal random variable

user135172 Let $\overline X$ be a binormal random variable with distribution $N_{\overline X}(\overline m, \Sigma)$ where, $\overline X = \left( \begin{array}{c} x \\ y \end{array} \right)$, $\overline m = \left( \begin{array}{c} m_1 \\ m_2 \end{array} \right)$ $ \Sigma = \left( \begin{array}{cc...

Question contains thank you.
0
Q: Intersection of two languages

Mithlesh UpadhyayLet $L=L_1∩L_2$, where $L_1$ and $L_2$ are languages as defined below: $L_1=\{a^mb^mca^nb^m∣m,n≥0\}$ $L_2=\{a^ib^jc^k∣i,j,k≥0\}$ Then $L$ is Not recursive Regular Context free but not regular Recursively enumerable but not context free. My attempt : $L_1$ is CSL(context sensitive langua...

Short title. Question contains please.
0
Q: Is $F$ is uniformly continuous?

learning mathsLet $f:\mathbb R\to \mathbb R$ be such that $\int _{-\infty}^{\infty} |f(x)|dx<\infty $. Define $F:\mathbb R\to \mathbb R$ by $F(x)=\int _{-\infty}^x f(t) dt$ Does it follow that : $1.f$ is continuous ? $2.F$ is uniformly continuous ? $3.|f|<M$ for some $M>0$? I think $1$ is false since we ...

Question contains please.
0
Q: Probability of correct forecast in a football match.

Adesh TamrakarThe question is : A football match may either won, drawn or lost by the host country's team. So there are three ways of forecasting the results of any one match, one correct and two incorrect. Find the probability of forecasting at least three correct results for four matches. I calculated it as...

Question contains please.
0
Q: Brain block! Help please (:

Lfg924x^3+21x^2-18x=0 Need a step by step because I'm clearly messing up along the way of solving this ! Thanks!

Welcome to Math.SE, Lfg92. Words such as help, please do not add information to titles. Please edit the title so that it better describes the specifics of your question. Do not hesitate to make it longer or include a formula if needed. More tips here. (autocomment)Normal Human 21 secs ago
0
Q: Prove $SL(n,\mathbb C)/B\cong SU(n)/T$

RonaldHow t prove that $SL(n,\mathbb C)/B\cong SU(n)/T$, where $B$ is the subgroup of upper triangular matrices and $T$ is the subgroup of the diagonal $n×n$-matrices?

Short question.
0
Q: analytic-complex analysis

Dan CastroGood day, please a dude how to prove this: $$Suppose\ f\ analytic\ and\ {f}'(z)\neq 0\ in\ a\ region\ \Omega.\ If\ z_{0}\ \in \Omega\ and\ f(z_{0})\neq 0, show\ that\\ for\ all\ \varepsilon > 0,\ there\ are\ z_{1},z_{2}\ such\ that\left | z-z_{0} \right |< \varepsilon,\left | z-z_{2} \right |< ...

Short title. Question contains please.
 
7:32 AM
0
Q: Slope of a harmonic map

MaryamLet $C\subset\mathbb{C}$ be an analytic arc and $p\in C$. For a harmonic map $f$ defined on a neighborhood of $p$, I want to find a formula for the slope of $f$ in $p$. Any help need. Thanks.

Short title. Short question.
0
Q: Distinctions of different topologies on the sequence space (countable cartesian products of $\mathbb{R}$)

Daqian Sun$\newcommand{\b}[1]{\mathbf{#1}}$ $\newcommand{\R}{\mathbb{R}}$ $\newcommand{\N}{\mathbb{N}}$ Question I solved this exercise in Munkres.(20.4) But I don't know if I did it righ t or not. I really appreciate it if anyone can take a look at it and give me some pointers. Problem Consider the pro...

Tagged proof-verification.
0
Q: Proving Equivalence of two structures

frogeyedpeasSo I was working on showing that $$\text{Th}(\Bbb{R}, 0,1,+, \le) = \text{Th}(\Bbb{Q}, 0,1,+, \le) $$ My Initial ideas for working on this problem was to systematically start by showing: $$\text{Th}(\Bbb{R}, 0,1,+) = \text{Th}(\Bbb{Q}, 0,1,+) $$ And $$\text{Th}(\Bbb{R}, 0,1,\le) = \text{...

Tagged proof-writing.
0
Q: how to prove that the function is bounded

Amneet SinghShow that (e^(-x))×(x^n) is bounded on [0,infinity) for all positive integral values of n. Using this show that integration of above function exists from 0 to infinity???

Short question.
Short title.
0
Q: Inequality with logs

ThePr0MathDudeLet $n>1$ be a integer and let $\lfloor\dfrac{n-2^{k}}{2^{2k+1}}\rfloor=0$. Show that there exists a constant $t$ such that $$\sum_{j=0}^{k}\left(\lfloor \dfrac{n-2^{j}}{2^{2j+1}}\rfloor+1\right)-\frac{4n}{9}\le t\log_{10} n .$$

0
Q: Question about a proof in Zorich's *Mathematical Analysis*

Math1000For context, what he means by a "base" is a basis for a topology on $X$, and the oscillation of $f$ over $B$ is $\omega(f,B)=\sup_{x_1,x_2\in B}|f(x_1)-f(x_2)|$, i.e. the diameter of the image of $B$ under $f$. Now, in this theorem: I do not see any reason why $\varepsilon/3$ was used to obtai...

Title contains question.
0
Q: Find the roots of the quadratic equation

user290098(M-1)x^2+2(m+3)x+(m-7)=0 Please find a relation between x' and x" independent from m then deduce the double roots of this equation . Please solve!!

Short question. Question contains please, please.
0
Q: 2^i - 2293 is always composite?

a boyIs $2^i - 2293$ always composite for i=1,2,3,...? I have known: if $2^i - 2293$ is prime, i must have the form $i = 24 k+1$ In[2]:= Table[FactorInteger[2^i - 2293], {i, 1, 241, 24}] Out[2]= {{{-1, 1}, {29, 1}, {79, 1}}, {{173, 1}, {193943, 1}}, {{6737807, 1}, {83550917, 1}}, {{399550573, ...

Short question.
0
Q: Hint on solving an integral

userHow can i solve this integral: $$\int\dfrac{4(f'+x f^{2})}{1-2x^{2}f} . e^{\int{\frac{4x(x f'+f)}{1-2x^{2}f}}dx} dx$$ where $f$ is a differentiable function of $x$. I would be happy to get some hints.

Short title.
0
Q: An integral equality $\exists \xi \in [a,b] \Rightarrow g(\xi)\int_a^\xi f(x)\text{d}x=f(\xi)\int_\xi^b g(x)\text{d}x$

LauraAssume $f(x),g(x)$ is continuous on $[a,b]$. show that there exists $\xi \in [a,b]$, such that $$g(\xi)\int_a^\xi f(x)\text{d}x=f(\xi)\int_\xi^b g(x)\text{d}x$$ I tried to use intermediate value theorem to $F(t) = g(t)\int_a^t f(x)\text{d}x-f(t)\int_t^b g(x)\text{d}x$. but I failed to find two...

Consider replacing (analysis) with a more specific tag for the relevant branch of analysis. (autocomment)Normal Human 21 secs ago
0
Q: permutation/combination

bogumbikerThere are six cards with numbers written on them: 1, 2, 3, 4, 5, 6 . Two of them are drawn at at time and put together to form fractions, e.g. 4/5 How many proper fractions can be formed? What is correct equation to solve this type of questions? thanks

Short title.
0
Q: Minimize/reformulate sum of products as convex problem

pyroIs it possible to optimize this objective function as such or transform it into an convex formulation? The unknown continuous variables $x_i \in [-1, 1]$ are nodes in a graph and for each edge $(i,j)\in E$, the goal is to maximise the difference between each nodes $x_i$ and $x_j$. I formulated ...

Title contains problem.
0
Q: $lim inf |2^m - 3^n|$

a boyIn order to make it clear, I ask three questions: Does $ |2^m - 3^n|<10^6 $ have any integers solution for m>20 ? $ \lim inf |2^m - 3^n|$ is infinite ? $ \lim inf |2^m - 3^n|/m$ is finite ?

A title should not be all-MathJax; having some plain text helps with search and navigation. (autocomment)Normal Human 21 secs ago
0
Q: Why is $\gcd(p^2 + 1,p^3 + 1) = 2$?

mickLet $p$ be an odd prime. Why is $\gcd(p^2 + 1,p^3 + 1) = 2$ ?

Short question.
0
Q: Let $a_1, a_2, a_3...$ be the sequence of all positive integers relatively prime to 75. Find the value of $a_{2008}$.

WilliamKinLet $a_1, a_2, a_3...$ be the sequence of all positive integers relatively prime to 75, where $a_1<a_2<a_3...$ with $a_1=1, a_2=2, a_3=4, a_4=7$. Find the value of $a_{2008}$. What I have done: If ${a_n}$ is relatively prime to 75, then it is relatively prime to 3 and 5. Consider the following ...

Question contains thank you.
0
Q: If $L(x1)$ = $L(x2)$, does $x1$ = $x2$?

ChrisReviewing for an exam, and the book says False. But I can't seem think of a way to prove this wrong...

Short question.
0
Q: partition inducing equivalence

amrita kunduas per the nptel lecture 23 on discrete mathematics https://www.youtube.com/watch?v=uOE7rSKTDbo&list=PL0862D1A947252D20&index=23 , the professor proves that every partition induces equivalence. but is it necessary that the elements in the partition blocks are necessarily reflexive symmetric and t...

Short title.
0
Q: $L^p$ spaces inclusion

Thomas YangShow that for any measurable function $f$ in a measure space, we have: $$ ||f||_p \leq \max\{||f||_r ,||f||_s \} $$ whenever $0<r<p<s$. Now by splitting the integrals into parts where $f>1$ and $f\leq 1$, we could easily show that $$ ||f||_p \leq ||f||_r+||f||_s \ $$ But I have no idea how to sho...

Short title.
 
8:40 AM
0
Q: Maximum possible elementa in a subset given a condition

ThePr0MathDudeLet $B$ be a subset of $A$ such that no element in $B$ is twice the other. Find the maximum number of elements possible in $B$ if $A=\{1,2,...,n\}$.

Short question.
0
Q: Is there a commone tangent between the graph of the two following function :$f(x)=x²$ and $g(x)= lnx$??

zeraoulia rafikI ask how do i show if there is a commone tangent between the graph of the two function $f(x)=x²$ and $g(x)= lnx $? Thank you for any kind of help

Title contains ??. Short question. Question contains thank you.
 
8:55 AM
0
Q: Problem on Bivariate normal distribution

Zhanfeng LimLet $X_1$ and $X_2$ have a bivariate normal distribution with parameters $\mu_1 = \mu_2 = 0$ and $\sigma_1 = \sigma_2 = 1$ and $\rho = 1/2$ Find the probability that all the roots of $X_1x^2+ 2X_2x + X_1 = 0$ are real. Hint: First establish that $X_1 + X_2$ and $X_1 - X_2$ are bivariate normally...

Title contains problem.
 
9:09 AM
0
Q: What is the relationship between Sharp-P and FP?

WieshawnIf I haven's misunderstood the meanings of Sharp-P and FP, I think: FP is the class of functions computable in polynomial time. #P is the class of functions that count the number of polysized certificates which return yes after we verifies them in polynomial time for instances of an NP p...

Question contains please, in advance.
0
Q: exponential functions with constant

ModjularI'm in a pre-calc class, and we're looking at logarithms and exponential functions. One of the exercises I'm struggling with is: 5e^2x = 6 + 29e^x I would ususally multiply each side by a log value to get those exponents by themselfs, but that 6 is throwing me off. How do I get rid of it? Or do...

Welcome to Math.SE, Modjular. This site uses MathJax formatting of formulas. More tips here. (autocomment)Normal Human 22 secs ago
 
9:22 AM
0
Q: Reasoning, order of age

Mithlesh UpadhyayHari(H), Gita(G), Irfan(I) and Saira(S) are siblings (i.e., brothers and sisters). All were born on 1st January. The age difference between any two successive siblings (that is born one after another) is less than three years. Given the following facts: Hari's age + Gita's age > Irfan's age + S...

Short title. Question contains please.
0
Q: limit of the $ (\sum_i x_i^\eta)^{1\eta}$

mannixIs this formula $\lim_{\eta \rightarrow \infty }(\sum_{i=1}^n x_i^\eta )^{\frac{1}{\eta}} = max(x_i)$ right ? where $\{x_i\}$ are real bounded numbers

Short question.
0
Q: Combinatorics Counting

WongsakornUse combinatorics to count how many ways can 25 identical pens be distributed to four students so that each student gets at least three but no more than seven pens. What I have done so far is look like it make sense but it doesn't work out I was thinking about view it as bit string of 0 and 1....

Short title.
0
Q: CFG for this language

user290220Give a CFG for the language $\{x \in \{a,b\}^* | x \not= ww \text{ for some w} \in \{a,b\}^*$ In my attempt to do this I understand odd length strings are automatically in the language, but don't know how to handle even length strings. What would the CFG look like? I know it starts with S-> E|U ...

Short title.
0
Q: Proving or disproving statements about operations with integers

LarsI'm really stuck with this one and I'm thankful for any help. Consider the following operations on the set of integers: $\hspace{8em} a\star b := a^2 + b^2 \hspace{5em} a\diamond b := a+b+2ab$ Prove or disprove the following statements: a.) The relation is associative b.) The relation is com...

Tagged proof-writing.
 
9:45 AM
0
Q: Reviewers Disagreeing about Reviews

MrsEdTLDR What procedures are recommended when disagreeing with a another user's review? forgive me if there is a dup case study I have a ping that leads me to this question and the comment below: LinearLayout with style and without orientation issue? It is about a suggested edit I rejected ...

 
0
Q: the fundamental theorem on affine mapping?

Godgog ArsenalEvery bijection between 2 affine spaces with the same dimension sending 3 collinear points into 3 collinear ones (in an 1-dimensional affine subspace) must be an affine mapping?

Short question.
0
Q: A simple combinatorics / counting question

Ice ArtI have this question for a sample quiz, I don't know any other way to answer it as it's not explained in the book. The question is: "In a faculty, there are 1300 students. Of these students, 300 are taking a unit in mathematics, 500 are taking a unit in physics. There are 150 students tak...

Words such as question do not add information to titles. Please edit the title so that it better describes the specifics of your question. Do not hesitate to make it longer or include a formula if needed. More tips here. (autocomment)Normal Human 25 secs ago
0
Q: Find a subset of columns to maximize the number of rows whose sum of entries in selected columns is equal or larger than a given number

mr nonameGiven a real m*n matrix A (m rows, n columns), a real positive number b. Each entry in A can be positive or negative. Find a subset of columns to maximize the number of rows whose sum of entries in those selected columns is at least b. The equivalent mathematical formulation: $$ \max_{x\in\{0,1...

Question contains please.
0
Q: Determine the function of f(z) : Singularities and Residue

MaximoffIs anybody can help me by explained to me step by step how to solve this question. The function f(z) has a double pole at z=0 with residue=2 and a simple pole at z=1 also with residue=2. It is also analytic at all other finite points of the plane and is bounded as |z| -> infinity. Also f(z=2)=5 ...

Welcome to Math.SE, Maximoff. This site uses MathJax formatting of formulas. Consider adding a tag for a broader subject area to which the question belongs. Some of these tags might fit. More tips here. (autocomment)Normal Human 28 secs ago
0
Q: Why isn't the derivative of mod(2x^2-3x), mod(4x-3)?

stackoverflow___acc1I don't quite understand why this is the case? Since when differentiating |2x^2-3x| you get ((2x^2-3x)(4x-3))/|2x^2-3x|.. when its 2x^2-3x, the derivative is 4x-3 and when its -(2x^2-3x) its -(4x-3)? |4x-3| = +/- (4x-3)? I think I might have understand something wrong here but I am not sure what....

0
Q: Prove that $R^{\omega}$ with the box topology has the following property

IllustionistProve that $R^{\omega}$ has infinitely many components in the box topology.

Short question.
0
Q: What does it mean? (set theory)

Martin PfefferWhat does the following notation mean, see "(i)":

Tag (notation) should not be the only tag a question has. Please add a tag for a subject area to which the question belongs. (autocomment)Normal Human 25 secs ago
0
Q: $c_0$, the space of sequences converging $0$ is complete with dual $\ell^1(\mathbb{N})$

Anthony PeterLet $c_0$ be the space of all complex sequences $(a_n)$ such that $$\lim_{n \to \infty} |a_n| =0$$ with norm $\|(a_n)\|_{c_0} = \sup_{n} |a_n|$. Is it fair to say that: Let $\{(a_n)\}_{n \in \mathbb{N}}$ be a cauchy sequence in $c_0$. Let $\epsilon >0$. Then there exists $N$ such that for $n...

Question contains please, please. Tagged proof-verification.
0
Q: How to solve lim as x approaches infinity for tanhx^x

CherryI got as far as lim x approaches infinity for lny=xlntanhx. I'm not sure what to do there. I know tanhx as x approaches infinity is one but 1^infinity isn't the correct answer.

Short question.
0
Q: Physics vs. Mathematical ZFC models

Raúl Aparicio Bustilloare the physical equations results independent (Schrödinger equation, statistical mechanics, etc... of the ZFC model "used"?

Short question.
0
Q: Simplify easy disjunctive normal form(DNF).

MoontegoSo, I am struggling with the last part of my expression, where I got the Expression . I made a Karnough map and it clearly shows that the last (x_2 inversion, x_3, x_4 inversion) is not needed at all, but I have no idea no how get rid of it.

Title contains easy.
0
Q: Fractional power

MuzamilBy the definition of square root we can take square root of 16 as 4,-4 but in problems why we take 16^(1/2) as 4? And also how can we say,If a^m=b^n then a=b(m/n)? because when we take x^2=a we solve it as x=+-{a^(1/2)}.

0
Q: Find the green function for the following BVP by using dirac delta function and solve the BVP using Green function

Jerome Cheang Yue LiangHere's the question: y" + y = x^2 + 1, y(0) = 5, y(1) = 0 I manage to get a solution to be this: y = -6cot(1)sin(x) + 6cos(x) + x^2 - 1 - 2cos(1)sin(x) + 2cot(1)sin(1)sin(x) Can somebody help me check if my yp is correct? Thanks!

This site uses MathJax formatting of formulas. Questions tend to get more attention when they have a tag for a broad area of mathematics relevant to the question. Some of these tags might fit. More tips here. (autocomment)Normal Human 21 secs ago
 
10:54 AM
0
Q: Geometry problem involving orthocentre and midpoint of sides.

Gerard Let $AA_1, BB_1, CC_1$ be the altitudes of $\Delta ABC$ and let $AB \neq BC$. If $M$ is midpoint of $BC$, $H$ the orthocentre of $\Delta ABC$ and $D$ the intersection of $BC$ and $B_1C_1$, prove that $DH \perp AM$ I was trying to come up with a pure geometric proof but haven't got far. These...

Title contains problem.
0
Q: how to show that some nested radicals simplify to other

Hamzashow that (8^.5)*(9-(77)^.5)^.5 is same as 2(11)^.5-2(7)^.5 ^.5 means square root of the (). I have tried multiplying the radicals but that didn't work. the resulting radicals do not add up or get subtracted. I have tried taking commons also but that also doesn't work

Welcome to Math.SE, Hamza. This site uses MathJax formatting of formulas. Consider adding a tag for a broader subject area to which the question belongs. Some of these tags might fit. More tips here. (autocomment)Normal Human 22 secs ago
0
Q: Sum of $\sum 1/p^p&

spliblibA very simple question, but I can't seem to find anything relating to it : Is there any research, are there any results that have focused on or given insight on $\sum 1/p^p$, ${p \in \mathbb P}$ ? A very basic series, converges extremely fast, its value is around .29. What more can there be s...

Short title.
0
Q: Questions about vector fields on the upper half plane.

LJRI am reading the lecture notes. On page 46, why $R_{X}$ as a vector field on $\mathcal{H}$ is $L_{pXp^{-1}}X$? Why $R_{\kappa} = 0$, $R_{\alpha}=2y \frac{\partial }{\partial y}$, $R_{\nu_+}=y\frac{\partial }{\partial x}$? Thank you very much.

Question contains thank you.
0
Q: Any answer, spectral measure?

hichamgaussi want just to uhnderstand why in the hilbert space $$l^2(Z)$$ we have the spectral measure verifie $$\mu^x=1/2(\mu^x_{e_{1}}+\mu^x_{e_{2}})$$ where $$e_{1}(n)=\delta_{1,n} $$and $$e_{0}(n)=\delta_{0,n}$$ this what we use in the schrodinger operator

Consider replacing (analysis) with a more specific tag for the relevant branch of analysis. (autocomment)Normal Human 22 secs ago
0
Q: Tangent line to curve how to find if only the curve is known?

artinyHy I wonder how to find the tangent line ( at inflex point ) to curve...if I doesnt have function, only the measurement of the fuction / t-time,y - values/...so I have the plot of the function. for example: http://www.polymath-software.com/Pol...atlabGraph.jpg thank you /I want to in Matlab for...

Question contains thank you.
0
Q: Continue of parabolic equation

lanse7pty$\partial_tf(x,t)=\frac{\partial ^2f}{\partial x^2}$ $f(x,0)=g(x)$ $(x,t)\in\Omega\times(0,T]$,$\Omega$ is a bounded open subset of $R^n$. g(x) is smooth function,then, I want know ,how to show that $f(x,t) $ is continue about $t$.And $\Delta f$ is continue about $t$. And $f(x,t)$ is smooth ab...

Tagged pde, differential-equations.
 
11:12 AM
-3
Q: Forum-like alternative to StackOverflow

LittleByBlueSometimes I am facing issues I cannot solve on my own (just as everybody else on SE) but they are often not generic, so they do not fit really into the question pattern of SE. Therefore I would rather ask them in some kind of forum, than spamming SO with my questions. But I just tried to use t...

 
11:26 AM
0
Q: find a real number k so that ln(n) = n^k for all n

GravityI am trying to find the real number $k$ so that $$ln(n) = n^k$$. I know it is a number between $1/e$ and $1/2e$ but I am not sure how I could find the number.

Short question.
0
Q: The value of $m$ for which straight line $3x-2y+z+3=0=4x-3y+4z+1$ is parallel to the plane $2x-y+mz-2=0$ is

diyaThe value of $m$ for which straight line $3x-2y+z+3=0=4x-3y+4z+1$ is parallel to the plane $2x-y+mz-2=0$ is $(A)-2\hspace{1cm}(B)8\hspace{1cm}(C)-18\hspace{1cm}(D)11$ My Attempt:The plane $2x-y+mz-2=0$ is parallel to the $3x-2y+z+3=0$ and $4x-3y+4z+1=0$. So the lines are perpendicular to the n...

Question contains please.
0
Q: Hasse diagram, find: max, min

user288083We have got set of students in lecture room. Every student is in relation with yourselft, student X and student Y are in relation, if X is on left of Y (from view of teacher). Relation R: $(X,X) \in R$ $(X,Y) \in R \wedge (Y,X) \notin R$ $(X,Y) \in R \wedge (Y,Z) \in R \wedge (X,Z) \in R $ ...

Short title.
 
2
Q: Which are blacklisted tags and blacklisted phrases?

Martin SleziakThere are many tags which are blacklisted. And so are some strings/words/phrases in the title. Is it possible to have a list of them publicly available somewhere? As I was told by a moderator in chat: "Some of the blacklisted input is more sensitive (some stuff that helps prevent certain type...

 
0
Q: An informal account of Godement resolution??

Javier Ariaswhile reading on the Godement resolution regarding stalks of a sheaf and the relation to the whole of the sheaf itself, https://en.wikipedia.org/wiki/Godement_resolution I found this statement: It allows one to view global, cohomological information about the sheaf in terms of local informati...

Title contains ??. Question contains in advance.
0
Q: Total Probability Rule

Mr AssistanceI have been given the following information: Change in sequential positive information next quarter 0.55 Change in sequential negative information next quarter 0.45 Change in sequential positive information in prior quarter 0.55 Change in sequential negative information in prior quar...

Short title.
0
Q: Perfect square and prime number

SylvainHello is anyone can help me with this problem: A non zero natural number N is such as N(N+2013) is a perfect square. Show that N can not be a prime number Find a N value such as N(N+2013) is a perfect square I've tried to proceed using a proof by contradiction) assuming that N(N+2013) is a ...

Welcome to Math.SE, Sylvain. This site uses MathJax formatting of formulas. More tips here. (autocomment)Normal Human 22 secs ago
0
Q: programming a heat problem

obua innocentDevelop the finite element equilibrium equations for a heat transfer problem and then develop a computer programme for its solution. Use a FOUR isoparametric finite element grid to test your programme with the domain below. given that the centre of the inner cylinder is 2.5cm from the centre of t...

Welcome to Math.SE, obua innocent. Consider adding a tag for a broader subject area to which the question belongs. Some of these tags might fit. (autocomment)Normal Human 22 secs ago
0
Q: What does dz^2 mean?

CassI'm reading a paper ("La Formule de Verlinde" by Christoph Sorger) and at a certain point, the author switches from algebra geometric language to complex geometric language. He uses the symbol $dz^2$, where $z$ is a local complex coordinate on a Riemann surface. Can anyone explain what this means...

Short title.
0
Q: How to solve $y′+2xy=2x^3y^3$?

Андрей ТоцкийI am trying to solve. $y′+2xy=2x^3y^3$? Help me please-_-

Short question. Question contains please.
0
Q: Methods for finding solution to $A\mathbf{x}=b$ problem

ELECLet $A=PDP^{-1}$ where $P = \begin{bmatrix}1&0&0\\0&2&1\\2&-5&-3\end{bmatrix}$ and $D=\begin{bmatrix}8&0&0\\0&0&0\\0&0&7\end{bmatrix}$. Find all solutions to $x = \begin{bmatrix}x_1\\x_2\\x_3\end{bmatrix}$ to equations $A\mathbf{x} = 8 \mathbf{x}$. I tried the brute force approach and calculated...

Title contains problem.
0
Q: Approximation of the central support of a projection with finite sum of smaller projections

John N.Let $(M,\tau)$ be a von Neumann algebra, i.e. a unital subalgebra $M=M''\subset \mathbb{B}(H)$ with a finite (faithful) trace $\tau: M\to \mathbb{C}$ (faithful means that $\tau(x^*x)=0$ implies $x=0$; moreover $\tau(q)\leq 1$ for any projection $q\in M$). Consider a projection $p\in M$. If $p$ i...

Question contains thank you, in advance.
0
Q: What is the correct approach for studying bifurcations?

user2983638Probably a trivial question. Let's say I have the following system of equations: \begin{cases} f\left(x,y\right)=0\\ \\ y=g\left(x\right) \end{cases} and I want to study its saddle-node bifurcations (where an eigenvalue goes to zero), and its Hopf bifurcations (where two eigenvalues have the fo...

Questions tend to get more attention when they have a tag for a broad area of mathematics relevant to the question. Some of these tags might fit. (autocomment)Normal Human 23 secs ago
0
Q: Khintchine inequality (question about Holder inequality)

krirkrirkI'm reading a proof of a Khintchine inequality : Let $(r_{1}, \dots , r_{n})$ be iid random variables with $P(r_{i} = > \pm 1) = \frac{1}{2}$. Let $f = \sum\limits_{j=1}^{n}a_{j}r_{j}$, where $a_{j} \in \mathbb{R}$. Then $||f||_{2} \leq \sqrt{e}||f||_{1}$. The proof uses Holder : $...

Words such as question do not add information to titles. Please edit the title so that it better describes the specifics of your question. Do not hesitate to make it longer or include a formula if needed. More tips here. (autocomment)Normal Human 29 secs ago
0
Q: A query in the definition of multivariable function's limit

karhasAt my multivariable calculus class we gave this definition for the limit of a function: Definition: Let $ \mathbb{R}^n \supset A $ be a open set , let $f:A \to\mathbb{R}^m $ be a function, let ${\bf x_0}$ be a point of $A$ and ${\bf P}$ a point of $\mathbb{R}^m$. To say that $f$ ...

Title contains query.
0
Q: Difference between Probability and Likelihood

LvergergskI am confused with the difference between Probability and Likelihood, because both of them use the same symbol. I have made a hypothetical setting to specify my question: If $X_2$ is only depends on $X_1$, $X_3$ is only depends on $X_2$, that is $(X_1)\to(X_2)\to(X_3)$. (just like markovian mod...

Question contains thank you.
0
Q: Whats the subject for measuring space?

DanielI'm trying to draw something and I need to know what subject in maths involves space like how much room there is in something

Short question.
0
Q: Function with compact support?

hichamgausswhy $$D(\overline\Omega)=C^\infty(\overline\Omega)$$ where $$D(\overline\Omega)$$ set of function with compact support in $$\Omega$$

Short question.
0
Q: How to combine percentiles from different dataset

user3311663I'm trying to combine percentiles (50th and 90th) from 2 different datasets. I don't have access to the datasets at the time of combining. Apart from the percentiles I have the number of elements on each dataset. Now, I know that any attempt at combining them will be only an approximation to the...

Welcome to Math.SE, user3311663. Consider adding a tag for a broader subject area to which the question belongs. Some of these tags might fit. (autocomment)Normal Human 32 secs ago
0
Q: The equation of the plane which passes through the point of intersection of two space lines and at greatest distance from the point $(0,0,0)$

diyaThe equation of the plane which passes through the point of intersection of lines $\frac{x-1}{3}=\frac{y-2}{1}=\frac{z-3}{2}$ and $\frac{x-3}{1}=\frac{y-1}{2}=\frac{z-2}{3}$ and at greatest distance from the point $(0,0,0)$ is $(A)4x+3y+5z=25\hspace{1cm}(B)4x+3y+5z=50\hspace{1cm}(C)3x+4y+5z=49\h...

Question contains please.
0
Q: Use fourier transform to solve wave equation

Jerome Cheang Yue LiangHere's the question. ∂^2/(∂t^2 ) u(x,t)=c^2 ∂^2/(∂x^2 ) u(x,t) We are supposed to use this form of fourier transform to solve our PDE f(hat)(s) = 1/√2π ∫_(-∞)^∞▒〖f(t) e^(-ist) dt〗 Can anyone enlighten me on how to do this question? Thanks!

 
12:49 PM
0
Q: Continuity of probability

ZimkovicLet $(B_t)_t$ be a Brownian motion, then I am given a stopping time $\tau_s:=\min(\inf\{t \ge 0; B_t=a\}, \inf\{t \ge s; B_t=b\}; \inf \{t \ge 0;B_t=c\}),$ where $a<0<b<c.$ Now, I want to show that $P(B_{\tau_s}=c)$ is a continuous function in $s.$ What I think is easy is to show that it is co...

Short title.
0
Q: Determining Invariant Elements under a Subgroup of the Galois Group

Sir JectiveI am currently reading J.P. Escofier's "Galois Theory" and in the text he discusses the galois group of $\mathbb{Q}(\sqrt[3]{2}, j)$ which is isomorphic to $S_3$. I have become lost in his discussion of finding the field of invariants of a given subgroup of the Galois group, particularly in refer...

Question contains in advance.
 
0
Q: This whole voting thing is silly

mickWith respect to all users of MSE but really , this voting thing is silly. Construct a function which is continuous in $[1,5]$ but not differentiable at $2, 3, 4$ This trivial question got +55 and an answer only containing W with more then 100 upvotes ! I get a bit annoyed when good questions a...

 
0
Q: Lebesgue measure of proper subset

 zemmahIs it true that if A is a proper subset of B, then the Lebesgue measure of A is strictly less than the Lebesgue measure of B?

Short question.
0
Q: Boundary point are differ from boundary point in a metric space. How?

Krishnapal SinghBoundary point are differ from boundary point in a metric space. How? Explain with sutaible examples.

Consider adding a tag for a broader subject area to which the question belongs. Some of these tags might fit. (autocomment)Normal Human 27 secs ago
 
1:12 PM
0
Q: ε–δ definition of a limit

Lauren BurkeI was wondering if anyone could help with this ε–δ definition of a limit. I have looked it up in my calculus book and online and I just don't understand how to do it. Prove, using the ε–δ definition of a limit that lim(x,y)→(0,0);(x,y)∈Df(x, y) exists where D ⊆ R^2 is the domain of f and f(x,...

 
1
Q: Question regarding downvotes on asked question

Mystriss FreyaI'm new here. I was having some trouble with a bit of code and I've read a lot of advice on here though web searches so I thought this might be a good place to come for help with my particular issue. However, it's not feeling that way because I got down voted, apparently because some other user ...

 
0
Q: showing $Ax=b$ has a unique solution by finding the fixed points of a function

user189013Let $n ∈ \mathbb N$. Consider an $(n×n)$-matrix A with real components and a column vector $b ∈ \mathbb R^n$. They give rise to an affine transformation $T : \mathbb R^n → \mathbb R^n$ with $T(x) = Ax+b$. Consider the Euclidean metric on $\mathbb R^n$: Let I be the identity $(n × n)$-matrix. Sup...

Question contains in advance.
0
Q: Moment Generating function hard example!

alessia123X is a random variable with density $$f(x)=2e^{-2x+2} , x\geq1$$ and 0 otherwise. Determine $Mx(θ)$, the moment generating function for X, and the values of θ for which $Mx(θ)$ is defined. Use $Mx(θ)$ to determine $E(X^2)$ Working: I know that $Mx(θ) = E(e^{\theta x})= \int_{0}^{\theta}e^{\th...

Title contains hard.
0
Q: Number of lines which are normal as well as tangents to the curve $y^2=x^3$?

Sanchayan DuttaNumber of lines which are normal as well as tangents to the curve $y^2=x^3$? What is the general method to solve such problems?I could'nt proceed much.

Short question.
0
Q: geometry question on quardilaterals

AyushIn a quad. ABCD with AB=CD,P and Q are mid points of diagonals AC and BD.P and Q joined and extended hits both sides AB and CD at S and T respectively.How can I prove that angle AST=angle DTS

Words such as question do not add information to titles. Please edit the title so that it better describes the specifics of your question. Do not hesitate to make it longer or include a formula if needed. More tips here. (autocomment)Normal Human 21 secs ago
0
Q: Why is the totient function expressed as $\phi(n)=n \prod_{p|n}\left ( 1-\frac{1}{p} \right )$

TrogdorThis seems a little inconvenient, and using $$\phi(n)=\prod_{p|n} \left ( p-1 \right )$$ seems much more convenient for computational purposes. So I am guessing that there is a specific reason for expressing it as such. What is that reason?

Consider adding a tag for a broader subject area to which the question belongs. Some of these tags might fit. (autocomment)Normal Human 21 secs ago
 
0
Q: Off-by-one error on Reputation tab on profiles

Joeytje50So yeah, Off-by-one bug in reputation graph legend and A new (?) off-by-one error on the reputation graph apparently either haven't been fixed yet, or are back again. Clicking on one of the bars on your reputation tab will give you the reputation history of the day before that. Is there any news...

 
1:39 PM
0
Q: how to simplify the nested radical?

M.Hamza Ali2^(1/2) * (9-77^(1/2))^(1/2) How to simplify this so that it has no nested radicals? This question is same as that already posted but with a different point of view.

This site uses MathJax formatting of formulas. Consider adding a tag for a broader subject area to which the question belongs. Some of these tags might fit. More tips here. (autocomment)Normal Human 25 secs ago
0
Q: Are binary predicates sufficient

dan bI seem to recall a result that says any set of axioms can be converted to a set of equivalent axioms that use only binary predicates and constants. Can anyone point me to that result?

Short question.
0
Q: Properties of a random walk

krirkrirkFirst of all, I know nothing about Markov chains, and I'd like to prove the following without using the theory around them. Let $(M_{n})_{n\geq 1})$ be a random walk over $\mathbb{Z}$, starting at $M_{0} = 0$. Let $$S_{a, b} = \lbrace M_{i} \neq 0, a \leq i < b\text{ and } M_{t} = 0 \rbrace$$ a...

Short title.
0
Q: Rudin Taylor theorem and Newtons Method. [Problem 5.25]

user160110The Problem Suppose $f$ is twice differentiaable on [a,b], $f(a)<0$, $f(b)>0$, $f'(x)\geq \delta >0$, and $0\leq f''(x)\leq M$ for all $x\in [a,b]$. Let $\xi $ be the unique point in $(a,b)$ at which $f(\xi)=0$. Choose $x_1\in (\xi, b)$, and define $\{x_n\}$ by $x_{n+1}=x_n-\frac{f...

Title contains problem.
0
Q: Taylor's Theorem expansion

Vadims GulkevicsI need to show that $f'(x) = (f(x-2h) - 4f(x-h)+3f(x)) / 2h +0(h^2)$ with Taylor series expansion of $f(x-h)$ and $f(x-2h)$. I got the expansions but I don't get the final answer correct, so I think I am missing something.

Short title.
0
Q: Proving $\|AB\| ≤ \|A\|\|B\|$

user189013I want to show that for all $A, B$ n by n-matrices with real components, $\|AB\| ≤ \|A\|\|B\|$. I know the proof of this inequality using two vectors. Is it the same for matrices as well? Any help and hint would be appreciated.

Short title.
0
Q: $2*sinA/cos3A + 2*sin3A/cos9A + 2*sin9A/cos27A=tan27A-tanA$

Kalpan$Prove that$: $2*sinA/cos3A + 2*sin3A/cos9A + 2*sin9A/cos27A=tan27A-tanA$

A title should not be all-MathJax; having some plain text helps with search and navigation. (autocomment)Normal Human 27 secs ago
0
Q: If $f(x) \in F[x]$ is irreducible over $F$, and let $E$ the splitting of $f(x)$.

Leonardo Francisco CavenaghiThen, supposing that $[E:F] = p^k$, for $p$ prime. Then $f$ is solvable by radicals. I can't think in a tower of roots to solve the problem. Help, please.

Short question. Question contains please.
0
Q: General question on Group Actions.

User9523A group $G$ acts on a set $A$.. With this statement we can say that : (1) There's a permutation from $A$ to $A$ as : $$ \sigma_g : A \: \: \:->\: \: A$$ $$ \sigma_g (a) =g \wedge a$$ where $g \wedge a$ says that $g$ acts on $a$. (2) The map form $G$ to $S_A$ defined by : $$ \phi(g) = \sigma_g...

Words such as question do not add information to titles. Please edit the title so that it better describes the specifics of your question. Do not hesitate to make it longer or include a formula if needed. More tips here. (autocomment)Normal Human 28 secs ago
0
Q: How to find the 999th shape in a pattern?

BenThe pattern is of a triangle followed by 3 stars and 4 circles and it continues to repeat. What will the 999th shape in the pattern be? Explain how you can tell.

Questions tend to get more attention when they have a tag for a broad area of mathematics relevant to the question. Some of these tags might fit. (autocomment)Normal Human 23 secs ago
0
Q: how to find N = x1^a1 * x2^a2 * upto xn^xy?

user139256Suppose there is any number N . N = x1^a1 * x2^a2 * x3^a3 and so on . We have to find x1,x2,x3,..... xn (all x are prime) and a1,a2,a3....an . How to find out N using minimum calculation ? e.g 16200 = 2^3 * 3^4 * 5^2 . x = 2,3,5 a = 3,4,2 .

 
2:20 PM
0
Q: Should I answer the question if someone has already answered it in a comment?

SweeperI came across a question: How to remove specefic rows from linq result I see that there is already a comment that has answered the question and OP has already "accepted" it with another comment. Should I provide a more detailed answer? I think I should. Because adding the answer does not count...

 
0
Q: semimartingale and limit

ploufLet $X_t=X_0+M_t+A_t$ a continuous semi-martingale. Let $g: \Bbb R \to [-1,1]$, of class $C^{\infty}$, with $g(x)= \left\{ \begin{matrix} -1, & x \le 0 \\ 1, & x \ge 1 \end{matrix} \right.$. Let $f_n : \Bbb R \to \Bbb R$ with $f_n(0)=0$ and $f_n'(x)=g(nx) \ \forall x \in \Bbb R$. Show $\lim \lim...

Short title. Question contains thank you.
0
Q: Asymptotic Complexity

Kirchoff_SvenI have to determine the value of truth(always true, always false, sometimes true) of the following statement: f(n)= Ω(n) and g(n) = o(n^2), find the value of truth of f(g(n)) = ω(n) I don't know eactly what I'm supposed to do. Any help will be much appreciated

Short title.
0
Q: Determine and prove the limit

Dandy DonFind and prove the limit as n tends to infinite for the following sequence: $$\sum_{n=0}^\infty z^n$$ Where z is a set of the complex numbers and $|z|<0.6$

Short title. Short question.
0
Q: Why $\partial_tT=(\delta_\dot gT)(g)=(\delta_hT)(g)$?

lanse7ptyWhy $\partial_tT=(\delta_\dot gT)(g)=(\delta_hT)(g)$? $h=\partial_tg,T$ is a $(k,l)-tensor$.

Short title. Short question.
 
2:42 PM
0
Q: Find the word at $48$ position?

justin takro The letters in the word "PLACES" are permuted in all possible ways and arranged in the alphabetical order.Find the word at 48 position. a)AESPCL b)ALCEPS c)ALSCEP d)AESPLC MyApproach As per dictionary I started with AC--->$4$!=$24$ AE--->$4$!=$24$ So,I think the word start with AE b...

Question contains please.
0
Q: Formulate flow optimization problem a LP problem

Mathguy007One sends a units of red flow from node 1 to node 9 and b units of blue flow from node 4 to node 6. The red flow can only go south and the blue flow east. The two flows down not mix. The load on a given link is defined as the sum of the sizes of red the blue flows and one would like to send the r...

Title contains problem, problem.
0
Q: Attractors in Duffing equation

Vaggelis_ZThe Duffing equation in its full form is $$\ddot{x} + \delta \dot{x} -ax + \beta x^3 = \gamma \cos(\omega t)$$ Now for specific values of the parameters several attractors exist (or not). Let's assume that $\alpha = \beta = \omega = 1$, $\delta = 0.15$, while $\gamma = 0.2445$. For these values...

Question contains in advance.
0
Q: Concerning the LU decomposition and Eigenvalues

almostdoneI've been asked to "fill in the blanks" on the following: Elimination produces A = LU . Te eigenvalues of U are on its diagonal; they are all the __________. The eigenvalues of L are on its diagonal; they are all _________. The eigenvalues of A are not the same as ____________. I know that the ...

0
Q: Minimum Hamming distance subvector

G HI can find hamming distance between $v \in U^n$ and $u \in U^k$, $k \le n$, $U$ - small alphabet(for example $B$ or $DNA = \{A,C,G,T\}$), beginning some $j$: $Hamming\ distance^j(u, v) := Hamming\ distance (u, (v_j, \ldots , v_{j + k - 1}))$, $1 \le j \le n - k + 1$. I want to find $i$ : $Hamm...

Question contains thank you.
0
Q: Simplify 4-term Boolean Algebra expression

JTJMHow do I get from this: $F = AB' + AC' + AD + C'D'$ to this: $F = AB' + AD + C'D'$ Not sure how the $AC'$ disappeared.

Short question.
0
Q: uniqueness discrete valuation

flawrGiven a ring R and the discrete valuations v,w such that (R,v) and (R,w) are discrete valuation rings, I have to show that u=v. Can anyone give me a hint on how to go about this? I have no idea where to start. Up to now i could show that valuation of units must be zero, and I suspect that I hav...

Consider adding a tag for a broader subject area to which the question belongs. Some of these tags might fit. (autocomment)Normal Human 22 secs ago
0
Q: Urgently need help!!!!!!( Annuity)

AndersonI have a problem that I cant figure out. how does the 2nd line become 3rd line. I need details explanation especially on the 1+i-1 of the 3rd line. Thanks

Welcome to Math.SE, Anderson. Words such as help do not add information to titles. Please edit the title so that it better describes the specifics of your question. Do not hesitate to make it longer or include a formula if needed. More tips here. (autocomment)Normal Human 22 secs ago
0
Q: proving that delta exists to a limit

NotSureThe question is : I know that because it is known that there is a limit i can choose an epsilon, so if i'll say epsilon = 1 then 4 < f(x) < 6. i also know that 1 - delta < x < 1 + delta yet i am not sure how that helps me. Thanks in advance !

0
Q: Finding the length of semi major axis of an ellipse given foci, directrix and eccentricity

user290002Can someone explain the solution to this. I don't get the assumptions in his solution. "A focus of an ellipse is at the origin. The directrix is the line x = 4 and the eccentricity is 1/2. Then the length of the semi−major axis is?" Solution: "Without loss of generality, let h > 0, let (h, 0) b...

0
Q: Basic Probability for Events

AnkitConsider sample space x = {v1,v2,v3,v4} and 3 Events E = {v1} F={v1,v2} G={v1,v2,v3} suppose their Probability are P(E) = 1/10 P(F) = 5/10 and P(G) = 7/10 Now define a fourth Event H = {v2,v4}.find P(H)

Short title. Title contains basic.
 
3:18 PM
-1
Q: Area 51, propsal disappeared

TungstenXI posted a proposal (Men @ 40) at Area 51 with two suggested questions. I thought the questions were relevant but it seems that others didn't both for -5 at the same time. Now it says 0 proposals in my profile and I can't find it any where. It would be nice if I was informed that it was canned.

 
0
Q: Tricky collinear vectors problem HELP!

dahaka5Let OABC be any planar quadrilateral. Let $G_1, G_2 and G_3$ be the centroids of OAB, OBC and OAC respectively, and let G be the centroid of the triangle $G_1G_2G_3$. Show that the points O, G and the centroid D of triangle ABC are collinear. I have no idea how to start this any help would be ap...

Welcome to Math.SE, dahaka5. Words such as help do not add information to titles. Please edit the title so that it better describes the specifics of your question. Do not hesitate to make it longer or include a formula if needed. More tips here. (autocomment)Normal Human 29 secs ago
0
Q: boolean SOP expression simplification: F(a,b,c,d) = (a+d)(a'b+c'd)(ac+bd)'

keromy answer that I have gotten is b'c'd + a' b d' however, the answer given to me was b'c'd can someone tell me whether I am correct

Short question.
0
Q: identification with dual of a hilbert space?

hichamgaussi know that we can identify $H$ with his $H^*$(dual) then if we have an other sub-space $V$ of$H$ then we will have $V\subset H=H^*\subset V^*$ can we identify here $V$ with $V^*$ then we will have V=H sure we can not but why ?

Consider replacing (analysis) with a more specific tag for the relevant branch of analysis. (autocomment)Normal Human 29 secs ago
0
Q: Integral $e^{-x^{2}}$

MadhuHow to integrate $e^{-x^{2}}$ ?$ When I used geogebra I got the answer as $\frac{1}{2}\sqrt{\pi}(erf)(x)$. What is $ (erf(x))$ ? How to arrive at this answer ?

Short title. Short question.
 
3:36 PM
0
Q: Use this userscript when you share a link

OokerI see that there are many links in comments that don't have link text, therefore hard for me to know if the link is worth to click. For example, this link: Entropy and the principle of least action You are hesitate to click on that, right? You are feeling to wasting time if you click on that, ar...

 
0
Q: Proving Theorem on Sub-Group

AtineshConsider the below theorem on Sub-Groups Theorem : Let $(G, \circ)$ be a group and $H$ be a non-empty finite subset of $G$. Then $(H, \circ)$ is a subgroup of $(G, \circ)$ if and only if $a \in H, b \in H \implies a \circ b \in H$ Proving Forward : If $(H, \circ)$ is a subgroup of $(G, \cir...

Short title.
0
Q: How do I calculate a set in a map ?

pinkpanther5I've got a task, where a set A and a map g were given. The task was to calculate g(A). And I don't know what to do exactly, so I'd appreciate if s.o. could explain it with an example or give me a link, where it's explained. For example let A = { x in IR | |x-3|= 2 } and g (x) = (x+2)/x. So if I...

Consider replacing (analysis) with a more specific tag for the relevant branch of analysis. (autocomment)Normal Human 28 secs ago
0
Q: Clash of Clans. What's the total time to train?

asdhkall about clash of clans game. I have total of 6 barracks. Two of them produces one barbarian each every 20 seconds, the another two barracks produces one archer each every 25 seconds, and the last two produces 1 minion each every 45 seconds. the capacity of army camp is 200. one archer has an ...

Welcome to Math.SE, asdhk. Consider adding a tag for a broader subject area to which the question belongs. Some of these tags might fit. (autocomment)Normal Human 25 secs ago
 
0
Q: First Posts Review Queue - Is this really a good audit?

JALI recently failed this code-only answer audit in the First Posts Review Queue. As I understand from previous meta posts, code-only answers do not warrant a NAA flag. The answer is in fact an attempt at an answer. Whether it is actually a useful answer will be determined by comments and up/down...

 
0
Q: (Convergent sequences) Did I prove this correctly?

ArthurLet $(a_n)$ be a bounded sequence and this inequality holds: $2a_n\leq a_{n-1}+a_{n+1}$ Prove, that $b_n=a_{n+1}-a_n$ converges to zero. My Proof: $2a_n\leq a_{n-1}+a_{n+1}$ $\iff2(a_{n+1}-b_n)\leq a_n-b_{n-1}+a_{n+1}$ $\iff a_{n+1}-b_n\leq\frac{1}{2}a_n-\frac{1}{2}b_{n-1}+\frac{1}{2}a_{n+1...

Tagged proof-verification.
0
Q: Forward-time, centered space evalaution of the heat equation: numerical stability and unique solution

user4437416I have a script of code which models a planetesimal that is accreted into a planetary atmosphere. In the code, I include the physics of frictional ablation and thermal ablation. Frictional ablation is governed by the coupled evolution of 5 equations and thermal ablation is governed by sublimattin...

Questions tend to get more attention when they have a tag for a broad area of mathematics relevant to the question. Some of these tags might fit. (autocomment)Normal Human 25 secs ago
0
Q: Quick question about the distribution of a function of a random variable

user118224I would be really grateful if someone could check what I have done here; it should be quick: Let $\Phi$ be a random variable taking values in $[0,\pi]$ with PDF $f(\phi)=\frac{1}{2}\sin\phi$. Define: $$h(\phi)=a\cos(\phi)+b$$ where $a,b$ are positive constants. I need to find the distribu...

Title contains question. Tagged proof-verification.
0
Q: Variant of linear regression using perpendicular distance instead of vertical

Gyro GearlooseNormally, linear regression asks for a pair of parameters m,b such that for a set of given points {x_i,y_i} the variance of y-m*x-b is minimized (this minimizes the distance in y-direction only). Instead, I would like to find a line y'=m*x'+b such that the pairs (x_i,y_i) have minimal perpendicu...

0
Q: Help with higher-dimensional derivatives

user308973Let $G: \mathbb{R}^n \rightarrow \mathbb R$ be given by $$G(z)= \frac{1}{2}|(I-A)z+Az_0 - v_0|^2 +f(Az-Az_0 +v_0),$$ where $A$ is an $n\times n$ matrix and $f$ is a smooth function on $\mathbb{R}^n$ and $z_0$ and $v_0$ are fixed vectors in $\mathbb{R}^n$. I want to calculate the derivatives $DG...

Welcome to Math.SE, Gyro Gearloose. This site uses MathJax formatting of formulas. More tips here. (autocomment)Normal Human 27 secs ago
 
00:00 - 16:0016:00 - 00:00

« first day (13 days earlier)      last day (561 days later) »